• Nenhum resultado encontrado

Análise Real Volume 2 (Elon Lages Lima) - (Solucionario) Carlos Alberto Pereira Dos Santos

N/A
N/A
Protected

Academic year: 2021

Share "Análise Real Volume 2 (Elon Lages Lima) - (Solucionario) Carlos Alberto Pereira Dos Santos"

Copied!
201
0
0

Texto

(1)

DEPARTAMENTO DE MATEMÁTICA CURSO DE MESTRADO EM MATEMÁTICA

EXERCÍCIOS DE ANÁLISE NO R

n

TURMA 2012\1

Prof.

o

Carlos Alberto Pereira dos Santos

(2)
(3)

1

Exercícios do Livro Análise Real vol.2

1.1 - Topologia do Espaço Euclidiano

1.1.1

O espaço euclidiano n-dimensional

Exercício 1

Se |u + v| = |u| + |v|, com u 6= 0 (norma euclidiana), prove que existe α ≥ 0 tal que v = α · u. Solução.

|u + v| = |u| + |v| ⇒ |u + v|2 = |u|2+ 2|u||v| + |v|2

⇒ hu + v, u + vi = |u|2+ 2|u||v| + |v|2

⇒ |u|2+ 2hu, vi + |v|2 = |u|2+ 2|u||v| + |v|2

⇒ hu, vi = |u||v|.

Tomemos o vetor w = v − hv, ui

hu, uiu. Como hu, vi = |u||v|, então temos que: hw, wi =  v − hv, ui hu, uiu, v − hv, ui hu, uiu  = |u| 2|v|2 − hv, ui2 |v|2 = 0 ⇒ v = hv, ui hu, ui. Onde hv, ui hu, ui = |u||v| |u|2 = |v| |u| > 0.

(4)

Exercício 2

Sejam x, y, z ∈ Rn tais que (na norma euclidiana) |x − z| = |x − y| + |y − z|. Prove que existe t ∈ [0, 1] tal que y = (1 − t)x + tz. Mostre que isto seria falso nas normas do máximo e da soma.

Solução. Chamando u = x − y e v = y − z , temos que |u + v| = |u| + |v|. Ora, mas pela desigualdade triangular |u + v| ≤ |u| + |v|, onde a igualdade ocorre se, e só se u = αv, para a lgum α ≥ 0 ∈ R, disto resulta que existe α ≥ 0 ∈ R tal que u = αv, isto é, x − y = α(y − z) ⇒ (1 + α)y = x + αz ⇒ y = (1+α1 )x + (1+αα )z, daí chamando t = 1+αα , temos que t ∈ [0, 1] e satisfaz y = (1 − t)x + tz.

Se tomarmos os pontos x = (1, 0), y = (0, 0) e z = (0, 1), é fácil ver que eles não são colineares mas satisfazem |x−z|S = |x−y|S+|y −z|S, portanto na norma da soma a afirmação não é verdadeira. Da

mesma forma os pontos x = (2, 0), y = (1, 0) e z = (0, 1/2) são um contra-exemplo pra afirmação se considerarmos a norma do máximo.

Exercício 3

Sejam x, y ∈ Rnnão-nulos. Se todo z ∈ Rnque é ortogonal a x for também ortogonal a y, prove que x e y são múltiplos um do outro.

Solução. Tem-se x 6= 0 e y 6= 0. Se x = y não há nada para demonstrar. Suponha x 6= y então o vetor y − hx, yi

|x|2 · x é ortogonal a x e, por hipótese, também é ortogonal a y e

assim  y, y − hx, yi |x|2 · x  =  x, y − hx, yi |x|2 · x  ⇒  y − x, y − hx, yi |x|2 · x  = 0. como y − x 6= 0, temos y − hx, yi |x|2 · x = 0 ⇒ y = hx, yi |x|2 · x, portanto y é múltiplo de x. Exercício 4

Se kxk = kyk, prove que z = 12(x + y) é ortogonal a y − x. (A medida de um triângulo isósceles é também altura).

(5)

Solução. 1 2(x + y), y − x = 1 2hx + y, y − xi = 12(hx, yi − hx, xi + hy, yi − hx, xi) = 12(hy, yi − hx, xi) = 12(|y|2− |x|2) = 0, como queríamos provar.

1.1.2

Bolas e conjuntos limitados

Exercício 1

Dados a 6= b em Rndetermine c, pertencente à reta ab, tal que c ⊥ (b − a). Conclua que para todo x ∈ ab, com x 6= c, tem-se |c| < |x|.

Solução. ab = {a + t(b − a); t ∈ R}

Como c ∈ ab ; c = a + t(b − a) onde t é tal que hc, b − ai = 0 ⇒ ha, b − ai + t|b − a|2 = 0

⇒ t = −ha, b − ai |b − a|2 .

Assim, c é completamente determinado. Por outro lado:

|c|2 < |c|2+ |b − a|2 = |c + (b − a)|2 = |a + t(b − a) + (b − a)|2 = |a + (1 − t)(b − a)|2 = |x|2 ∀x ∈ ab

com x 6= c.

Portanto, |c| < |x|, ∀x ∈ ab.

Exercício 2

Sejam |x| = |y| = r, com x 6= y (norma euclidiana). Se 0 < t < 1, prove que |(1 − t)x + ty| < r. Conclua que a esfera S(0; r) não contém segmentos de reta.

Solução. Seja xy o segmento de reta de extremos x e y. Então xy = {(1 − t)x + ty; t ∈ [0, 1]}. Temos que

|(1 − t)x + ty| = |x − tx + ty| = |x + t(y − x)| ≤ |x| + t|y − x| ≤ r + t|y − x| < r. Como S(0; r) = {x ∈ Rn; |x| = r}, vê-se facilmente que a esfera não contém segmentos de reta.

(6)

Exercício 3

Dados o conjunto convexo X ⊂ Rn e o número real r > 0, seja Br(X) =

[

x∈X

Br(x). Prove que

Br(X) é convexo.

Solução. Sejam a, b ∈ Br(X). Então existem x0, x1 ∈ X tal que a ∈ Br(x0) e b ∈ Br(x1),

portanto |a − x0| < r e |b − x1| < r.

Seja c um ponto do segmento ab , então c = (1 − t)a + tb, para algum t ∈ (0, 1), daí para este t tome xc = (1 − t)x0+ tx1· xc∈ X pois X é convexo. Além disso, temos:

|((1 − t)a + tb) − xc| = |((1 − t)a + tb) − ((1 − t)x0+ tx1)| = |(1 − t)(a − x0) + t(b − x1)| ≤ |(1 − t)(a − x0)| + |t(b − x1)| = (1 − t)|(a − x0)| + t|(b − x1)| < (1 − t)r + tr = r.

Logo, c = (1 − t)a + tb ∈ Br(X), e como c é um ponto arbitrário do segmento ab, segue que

ab ⊂ Br(X), portanto Br(X) é convexo.

Exercício 4

Prove que o conjunto X = {(x, y) ∈ R2; x2 ≤ y} é convexo.

Solução. Tomemos a = (x1, y1) e b = (x2, y2) ∈ X ⇒ x21 ≤ y1 e x22 ≤ y2. Seja z =

t(x2 − x1, y2 − y1) + (x1, y1) um ponto pertencente ao segmento que liga a e b. Temos que [(1 −

t)x1+ tx2]2 = (1 − t)2x12+ 2t(1 − t)x1x2+ t2x22.

Como (x1−x2)2 ≥ 0 ⇒ x21+x22 ≥ 2x1x2, daí [(1−t)x1+tx2]2 = (1−t)2x21+2t(1−t)x1x2+t2x22 ≤

(1 − t)2x21+ t(1 − t)(x21+ x22) + t2x22 = (1 − t)x21 + tx22 ≤ (1 − t)y1+ ty2, portanto X é convexo.

Exercício 5

Seja T : Rm −→ Rn uma transformação linear. Prove que se T 6= 0 então T não é uma aplicação

limitada. Se X ⊂ Rmé um conjunto limitado, prove que a restrição TX : X −→ Rnde T ao conjunto

X é uma aplicação limitada.

Solução. De fato, dado x ∈ Rm se |T (x)| = c ∈ R+ então |T (nx)| = nc > 0. Logo T não é limitada, pois R é um corpo arquimediano.

(7)

Seja X ⊂ Rm um conjunto limitado. Tomemos a norma da soma, e como X é limitado, existe K tal que |x| ≤ K, ∀x ∈ X. Temos x = x1e1+ · · · + xmem. Seja M = máx{|T (e1)|, · · · , |T (em)|}. Daí,

|T (x)| = |T (x1e1+ · · · + xmem)| = |x1T (e1) + · · · + xmT (em)|

≤ |x1||T (e1)| + · · · + |xm||T (em)| ≤ M (|x1| + · · · + |xm|) ≤ M · K.

Portanto T (X) é um conjunto limitado.

1.1.3

Conjuntos abertos

Exercício 1

Para todo conjunto X ⊂ Rm, prove que int.X é um conjunto aberto, isto é int.int.X ⊂ int.X. Solução. Tomemos x ∈ int.X ⇒ ∃ r0 > 0; B(x, r0) ⊂ X.

Afirmação : B(x, r0) ⊂ int.X.

Prova: De fato, seja y ∈ B(x, r0) e tomemos ε = r0− |y − x|. Então para todo x ∈ B(y, ε) temos

|x − x| ≤ |x − y| + |y − x| < r0 − |y − x| + |y − x| = r0 ⇒ x ∈ B(x, r0) ⇒ B(y, ε) ⊂

B(x, r0) ⊂ X, portanto y ∈ int.X, logo int.X é aberto.

Exercício 2

Prove que int.X é o maior conjunto aberto contindo em X, ou seja, se A é aberto e A ⊂ X então A ⊂ int.X

Solução. Seja a ∈ A, como A é aberto, ∃r > 0 tal que B(a; r) ⊂ A, e já que A ⊂ X, segue-se que B(a; r) ⊂ X, i.e., x ∈ int.X. Então A ⊂ int.X. Assim, int.X = [

Aλ⊂X

Aλ, com Aλ aberto.

Exercício 3

Dê um exemplo de um conjunto X ⊂ Rncuja a fronteira tem interior não vazio e prove que isto não seria possível se X fosse aberto.

Solução. Tomando X = Q ⊂ R, temos que a fronteira dos racionais são os reais, pois, dado x ∈ R, toda bola aberta centrada em x irá conter números racionais e numéros irracionais. Fato decorrente da densidade dos racionais em R.

Dado X ⊂ Rnaberto, temos que X = int.X ⇒ ∀ x ∈ X, ∃ ε > 0 tal que B(x; ε) ⊂ X ⇒ ∂X = ∅,

pois x ∈ ∂X se toda bola aberta centrada em x possuir pontos do interior de X e do complementar de X. Assim, nenhum ponto x ∈ ∂X é ponto interior.

(8)

Exercício 4

Seja πi : Rn −→ R a projeção sobre a i-ésima coordenada, isto é, se x = (x1, . . . , xn) então

πi(x) = xi. Prove que se A ⊂ R2 é aberto então sua projeção πi(A) ⊂ R também é um conjunto

aberto.

Solução. Consideremos (Rn, |.|

max) onde a bola aberta de centro a e raio r > 0 é dada por

B(a; r) =

n

Y

j=1

(aj − r, aj + r).

Seja A ⊂ Rnum conjunto aberto e ai ∈ πi(A), então existe a ∈ A tal que πi(a) = ai.

Como A é aberto, existe r > 0 tal que B(a; r) =

n

Y

j=1

(aj − r, aj + r) ⊂ A.

Então ai ∈ (ai− r, ai+ r) = πi(B(a; r)) ⊂ πi(A), donde segue que πi(A) é um conjunto aberto.

Exercício 5

Prove que toda coleção de abertos dois a dois disjuntos e não-vazios de Rné enumerável.

Solução. Tome em cada aberto A dessa coleção um ponto pertencente ao conjunto não-vazio A ∩ Qn. Como Qné enumerável o mesmo ocorre com o conjunto dos pontos escolhidos, a cada um

dos quais corresponde um único aberto da aberto da coleção, pois estes são disjuntos.

1.1.4

Sequências em R

n

Exercício 1

Dada a sequência (xk)k∈N em Rn, sejam N0 e N00 subconjuntos infinitos de N tais que N = N0 ∪ N00.

Se as subsequências (xk)k∈N0 e (xk)k∈N00 convergem para o mesmo limite a, prove que lim

k∈Nxk = a.

Solução. Dado ε > 0, existem k1, k2 ∈ N tais que k > k1, k ∈ N0 ⇒ |xk− a| < ε e k > k2, k ∈

N00 ⇒ |xk− a| < ε. Seja k0 = max{k1, k2}. Como N = N0∪ N00, segue que k > k0 ⇒ |xk− a| < ε.

Logo lim xk= a.

Exercício 2

Dada a sequência (xk)k∈N Rn, prove que as seguintes afirmações são equivalentes:

(a) lim kxkk = +∞

(b) (xk)k∈Nnão possui subsequências convergentes.

(c) Para cada conjunto limitado X ⊂ Rn, o conjunto N

(9)

Solução. (a) ⇒ (b)

Suponha que houvesse uma subsequência (xk)k∈N0 ⊂ (xk)k∈Nconvergindo para a. Então dado  = 1,

∃ k1 ∈ N tal que ∀ k ≥ k1, k ∈ N0 ⇒ |xk− a| < 1 ⇒ ||xk| − |a|| ≤ |xk− a| < 1 ⇒ |xk| <

1 + |a|. Em contrapartida, para  = |a| + 1, ∃ k2 ∈ N tal que ∀ k > k2 ⇒ |xk| > |a| + 1, pois

lim

k→ ∞xk = +∞. Daí se tomarmos k0 = max{k1, k2}, então ∀ k ∈ N

0tal que k ≥ k

0, temos por um

lado que |xk| < 1 + |a| e por outro lado |xk| > |a| + 1. Contradição! Portanto (xk)k∈N0 não admite

subsequência convergente. (b) ⇒ (c)

Suponha que (xk)k∈N não possui subsequências convergentes e que para algum conjunto limitado

X ⊂ Rn, o conjunto NX = {k ∈ N; xk ∈ X} seja infinito. Desse modo a sequência (xk)k∈Nx

é limitada, então pelo teorema de Bolzano-Weierstrass ∃ N0 ( infinito) ⊂ NX ⊂ N tal que (xk)k ∈N0

converge, ou seja , (xk)k∈N admite subsequência convergente. Contradição!

(c) ⇒ (a)

Admitindo (c), suponha que lim kxkk 6= +∞ ⇒ ∃ A > 0; ∀ k0 ∈ N, ∃ k > k0 satisfazendo

|xk| < A, e neste caso temos que o conjunto limitado X = {k ∈ N; xk ∈ B(0; A)} é infinito.

Contradição!

Exercício 3

Sejam A ⊂ Rnaberto e a ∈ A. Prove que se lim

k→∞xk = a então existe k0 ∈ N tais que k > k0 ⇒

xk ∈ A.

Solução. Como lim

k→∞xk = a ⇔ Dado ε > 0, existe k0 ∈ N tais que kxk− ak < ε quando k > k0

i.e. ∀ε > 0, xk ∈ B(a, ε) para k > k0.

Seja ε := |a − ∂A|/2, daí xk∈ B(a; ε) ⊂ A quando k > k0.

Exercício 4

Se a ∈ ∂X, prove que existem sequências de pontos xk ∈ X e yk ∈ Rn− X tais que xk, yk k→∞

−→ a. Vale a recíproca?

Solução. Como a ∈ ∂X, ∀ ε > 0 a bola B(a; ε) contém pontos de X e Rn− X. Assim, ∀ k ∈ N,

existe xk ∈ X e yk ∈ Rn− X com |xk − a| < 1/k e |yk − a| < 1/k. Pela denifição de limite de

sequências, segue que xk, yk k→∞

−→ a. Reciprocamente, se xk, yk

k→∞

(10)

k > k0 ⇒ xk, yk∈ B(a; ε). Como ∀ ε > 0 a bola B(a; ε) contém pontos de X e de seu complementar

então a ∈ ∂X.

1.1.5

Conjuntos fechados

Exercício 1

Para quaisquer X, Y ⊂ Rn, prove que X ∪ Y = X ∪ Y e X ∩ Y ⊂ X ∩ Y . Dê um exemplo onde

não vale X ∩ Y = X ∩ Y . Solução.

• X ∪ Y = X ∪ Y :

X ⊆ X e Y ⊆ Y , logo X ∪ Y ⊆ X ∪ Y . Como X ∪ Y é fechado, segue que X ∪ Y ⊆ X ∪ Y . X ⊆ X ∪ Y ⇒ X ⊆ X ∪ Y e Y ⊆ X ∪ Y ⇒ Y ⊆ X ∪ Y . Logo X ∪ Y ⊆ X ∪ Y .

Portanto, X ∪ Y = X ∪ Y . • X ∩ Y ⊂ X ∩ Y :

X ⊆ X e Y ⊆ Y , logo X ∩ Y ⊆ X ∩ Y . X ∩ Y é fechado e contém X ∩ Y , mas X ∩ Y é o menor fechado que contém X ∩ Y , portanto X ∩ Y ⊂ X ∩ Y .

• Exemplo onde não vale X ∩ Y = X ∩ Y :

Sejam a, b e c ∈ R tais que a < b < c. Então para X = (a, b) e Y = (b, c) podemos verificar que X ∩ Y = { b} 6= ∅ = X ∩ Y .

Exercício 2

Diz-se que o ponto a ∈ Rné valor de aderência da seqüência (xk)k∈N quando a é limite de alguma

subseqüência de (xk)k∈N. Prove o conjunto dos valores de aderência de qualquer seqüência é fechado.

Solução. Seja F = { conjunto dos valores de aderência da sequência (xk)}.

Tomemos a ∈ F ⇒ B(a; εk) ∩ F 6= ∅, ∀ εk= 1/k, k ∈ N.

Para ε1 = 1, tomemos a1 ∈ B(a, ε1) ∩ F . Como a1 ∈ F ⇒ (xk)k∈N∩ B(a1; ε1− |a − a1|) 6= ∅.

Seja xk1 ∈ (xk)k∈N∩ B(a1, ε1− |a − a1|).

Prosseguindo dessa forma, no i-ésimo passo teremos ai ∈ B(a; εi) ∩ F . Como ai ∈ F ⇒

(xk)k∈N∩ B(ai; εi− |a − ai|) 6= ∅. Tomemos xki ∈ (xk)k∈N∩ B(ai, εi− |a − ai|).

Os termos (xki)i∈N constituem uma subsequência de (xk)k∈N, além disso |xki − a| < 1/i, ∀ i ∈

(11)

Exercício 3

Prove que um conjunto A ⊂ Rné aberto se, e somente se, A ∩ X ⊂ A ∩ X para todo X ⊂ Rn. Solução.

(⇒) Seja a ∈ A ∩ X. Então a = lim xk, (xk) ⊂ X. ∃k0 tal que k > k0 ⇒ xk ∈ A. Portanto

xk ∈ A ∩ X. Logo a ∈ A ∩ X ⇒ A ∩ X ⊂ A ∩ X.

(⇐) Se A não fosse aberto, existiria um ponto a que não lhe seria interior. Mas, neste caso a ∈ A ∩ Rn− A ⊂ A ∩ (Rn− A) = ∅. Contradição.

Exercício 4

Se X ⊂ Rm e Y ⊂ Rn, prove que se tem X × Y = X × Y em Rm+n.

Solução. É óbvio que X × Y ⊃ X × Y . Como X × Y é o menor conjunto fechado que contém X × Y ⇒ X × Y ⊃ X × Y . Por outro lado se (x, y) ∈ X × Y ⇒ ∃ (xk) ⊂ X e (yk) ⊂ Y tais que

xk −→ x e yk −→ y. Daí (xk, yk) ⊂ X × Y e lim(xk, yk) = (x, y) ⇒ (x, y) ∈ X × Y .

Portanto X × Y = X × Y .

Exercício 5

Prove que X ⊂ Rné fechado ⇔ X ⊃ ∂X. Por outro lado A ⊂ Rné aberto ⇔ A ∩ ∂A = ∅. Solução.

(i) X ⊂ Rné fechado ⇔ X ⊃ ∂X.

De fato, X é fechado ⇒ X = X ⇒ ∂X = X ∩ Rn− X = X ∩ Rn− X ⊂ X. Então ∂X ⊂ X.

Reciprocamente, se ∂X = X ∩ Rn− X ⊂ X, então X = X, pois do contrario se x ∈ X e

x /∈ X ⇒ x ∈ X e x ∈ Rn− X então x ∈ X e x ∈ Rn− X ⇒ x ∈ ∂X ⊂ X, logo X é fechado.

(ii) A ⊂ Rné aberto ⇔ A ∩ ∂A = ∅.

De fato, se sabe que ∂A = ∂(Rn− A). Logo:

∅ = ∂A ∩ A = ∂(Rn

− A) ∩ A ⇔ ∂(Rn

− A) ⊂ Rn

− A ⇔ Rn− A é fechado ⇔ A é aberto.

Exercício 6

Sejam A, B ⊂ Rn conjuntos limitados disjuntos e não-vazios. Se d(A, B) = 0, prove que existe

x ∈ ∂A ∩ ∂B.

Solução. Se d(A, B) = 0 então existem sequências (xk) ⊂ A e (yk) ⊂ B tais que

(12)

é limitado. O mesmo vale para yk, pois B é limitado. Daí, a = lim yk. Logo, a ∈ A ∩ B. Como A e

B são disjuntos, não podemos ter a ∈ A e a ∈ B. Portanto, a ∈ ∂A ∩ ∂B.

Exercício 7

Prove que o fecho de um conjunto convexo é convexo.

Solução. Sejam a, b ∈ A. Então existem sequências (ak) e (bk) em A tais que a = lim ak e

b = lim bk. Como A é convexo, então fixando t ∈ [0, 1] temos que (1 − t)ak+ tbk ∈ A, ∀ k ∈ N.

Daí, lim((1 − t)ak+ tbk) = (1 − t)a + tb ∈ A. Portanto A é convexo.

Exercício 8

Prove que se C ⊂ Rné convexo e fechado então, para todo x ∈ Rn, existe um único x = f (x) ∈ C

tal que d(x, C) = |x − x|

Solução. (Existência): C é fechado e {x} é compacto ⇒ ∃ x ∈ C; d(x, C) = |x − x|.

(Unicidade): Se x ∈ C então f (x) = x e a unicidade é óbvia, pois ∀ x 6= x0, |x − x0| > 0 = |x − x|. Se x /∈ C, então suponha que exista outro x ∈ C; d(x, C) = |x − x| = |x − x| = r. Ora, desse modo temos que x e x ∈ S(x, r). Daí ∀ t ∈ (0, 1) tem-se que x(1 − t) + tx ∈ C e |x(1 − t) + tx − x| = |(x − x)(1 − t) + t(x − x)| < r = d(x; C). Contradição !

1.1.6

Conjuntos compactos

Exercício 1

Seja K ⊂ Rncompacto, não-vazio. Prove que existem x, y ∈ K tais que |x − y| = diam.K.

Solução. Por definição, temos que diam.K = sup{|x − y|; x, y ∈ K}. Tome a norma euclidiana. Pela definição de sup, dado ε > 0, existem x, y ∈ K tais que diam.K ≤ |x − y| + ε e ∀x, y ∈ K vale |x − y| ≤ diam.K.

Temos que existem sequências xk, yk ∈ K tais que diam.K = lim |xk− yk|. Como K é limitado, e

passando a subsequências se necessário, diam.K = lim |xk− yk| = |x0 − y0| onde x0, y0 ∈ K. Por

K ser fechado, segue que K = K e x0, y0 ∈ K.

Exercício 2

Se toda cobertura aberta de um conjunto X ⊂ Rnadmite uma subcobertura finita, então prove que X

(13)

Solução. (Limitado)

Suponha que X fosse ilimitado. Então pra nenhum k ∈ N, X ⊂ B(0; k). Daí neste caso teríamos que [

k∈N

B(0; k) é uma cobertura de X que não admite subcobertura finita, portanto X deve ser limi-tado.

(Fechado)

Suponha que X não seja fechado, então existe (xk) ⊂ X; xk −→ a /∈ X. Daí, para cada k ∈ N,

considere o aberto Rn\B[a; 1/k] = A

k. Então

[

k∈N

Ak é uma cobertura aberta de X que não admite

subcobertura finita, portanto X deve ser fechado.

Exercício 3

Seja (xk) uma sequência limitada em Rn que possui um único valor de aderência. Prove que (xk) é

convergente. Dê exemplo de uma sequência (não-limitada) não convergente que tem um único valor de aderência.

Solução. Seja a um valor de aderência de (xk). Se não fosse a = lim xk, existiriam ε > 0 e uma

infinidade de índices k tais que |xk− a| ≥ ε. Passando a uma subsequência, se necessário, teríamos

lim

k∈N0xk = b, com |b − a| ≥ ε, logo b 6= a seria outro valor de aderência. Quanto ao exemplo, basta

tomar xk = 0 para k ímpar e xk = k.ei se k é par.

Exercício 4

Se K ⊂ U ⊂ R com K compacto e U aberto, prove que existe ε > 0 tal que x ∈ K, y ∈ Rn,

|x − y| < ε ⇒ [x, y] ∈ U .

Solução. Inicialmente vamos tomar o conjunto Rn − U , o complementar de U no Rn. Esse

conjunto é fechado, pois seu complementar é aberto. Sabemos que K é compacto, ou seja, fechado e limitado, e Rn− U é fechado, então, pelo fato desses conjuntos serem disjuntos, existe a ∈ K e

b ∈ Rn−U onde a distância é atingida. Em outras palavras, |x−y| ≥ |a−b|, ∀ x ∈ K e ∀ y ∈ Rn−U .

Fazendo |a − b| = ε, temos que |x − y| ≥ ε, ∀ y ∈ (Rn− U ), donde B(x; ε) ⊂ U . Assim, ∀ x ∈ K

e ∀ y ∈ Rntais que |x − y| < ε, temos que y ∈ B(x; ε) ⊂ U . Portanto, [x, y] ⊂ B(x; ε) ⊂ U .

Exercício 5

Seja X ⊂ Rntal que, para todo compacto K ⊂ Rn, a interseção X ∩ K é compacta. Prove que X é

(14)

Solução. Seja a ∈ X, então existe uma sequência (xk) ⊂ X tal que a = lim xk.

Defina K = {xk; k ∈ N} ∪ {a}. K é compacto. Daí, por hipótese X ∩ K é compacto, em particular

X ∩ K é fechado. Como (xk) ⊂ X ∩ K, então a = lim xk ∈ X ∩ K, portanto pertence a X. Logo

X é fechado.

1.1.7

Aplicações contínuas

Exercício 1

Seja f : Rm −→ Rncontínua. Prove que as seguintes afirmações são equivalentes:

(a) Para todo compacto K ⊂ Rna imagem inversa f−1

(K) ⊂ Rmé compacta.

(b) Se xké uma sequência em Rmsem subsequências convergentes, o mesmo se dá com a sequência

f (xk) em Rn. (Ou seja, lim xk = ∞ ⇒ lim f (xk) = ∞.)

Solução.

(a) ⇒ (b) Suponha que f (xk) possui uma subsequência convergindo para o ponto f (x0). O

conjunto K = {f (xk); k ∈ N} ∪ {f (x0)} seria compacto, logo f−1(K) seria um compacto contendo

todos os xk∈ Rm e então (xk) possuiria uma subsequência convergente.

(b) ⇒ (a) Seja K compacto e suponha, por absurdo, que f−1(K) não seja compacto. Então, como K é fechado e f é contínua, temos que f−1(K) é ilimitada. Daí, seja (xk) ⊂ f−1(K) ∩ Rmuma

sequência sem subsequências convergentes (basta tomar uma sequência ilimitada em f−1(K) ∩ Rm)

⇒ f (xk) ⊂ K e portanto admite subsequência convergente. Contradição.

Exercício 2

Prove que um polinômio complexo não-constante p(z) = a0+ a1z + · · · + anzn, considerado como

uma aplicação p : R2 → R2, cumpre uma das (portanto ambas) condições do exercício anterior.

Solução. Ora para todo z 6= 0 em R2, temos que p(z) = zna0 zn + a1 zn−1 + · · · + an−1 z + an  . Tomemos |p(z)| = |z|n· a0 zn + a1 zn−1 + · · · + an−1 z + an e |zk| → +∞.

(15)

Ponha q(z) = a0 zn + a1 zn−1 + · · · + an−1 z + an. A sequência |q(zk)| é limitada, pois

0 < |q(zk)| z }| { a0 zn k + a1 zn−1k + · · · + an−1 zk + an ≤ a0 zn k + a1 zkn−1 + · · · + an−1 zk + |an| → |an|,

quando |zk| → +∞. Como |q(zk)| é limitada e lim |zk|n= +∞, tem-se que

lim |zk|n· |q(zk)| = +∞.

Exercício 3

Sejam X ⊂ Rm, K ⊂ Rncompacto e f : X × K → Rp contínua. Suponha que, para cada x ∈ X, exista um único y ∈ K tal que f (x, y) = 0. Prove que y depende continuamente de x.

Solução. Defina

g : X → K x 7→ y,

onde y é o único elemento de K que satisfaz f (x, y) = 0. Temos que g está bem definida. Resta provar que g é contínua. Para isto fixemos a ∈ X e tomemos (xk)k∈N; xk

k∈N

−→ a. Suponha que g(xk) não convirja pra g(a). Então existe  > 0 e infinitos índices k ∈ N; g(xk) /∈ B(g(a), ).

Tomemos N0 = {k ∈ N; g(xk) ∈ B(g(a), )}. Assim, (g(x/ k))k∈N0 ⊂ K ⇒ ∃ N00 ⊂ N0 e

b 6= g(a) ∈ K tal que g(xk) k∈N00

−→ b. Como f é contínua em X × K ⇒ lim

k∈N00f (xk, g(xk)) =

f (a, b) 6= 0, pois b 6= a e g(a) é o único elemento de K que satisfaz f (a, g(a)) = 0. Ora, mas f (xk, g(xk)) = 0, ∀ k ∈ N00, portanto se tomarmos  = |f (a, b)|/2, temos que ∃ k0 ∈ N tal que

∀ k ∈ N00, k > k

0 ⇒ |f (xk, g(xk)) − f (a, b)| < , e daí |f (xk, g(xk))| = |f (xk, g(xk) − f (a, b) +

f (a, b)| ≥ |f (a, b)| − |f (xk, g(xk)) − f (a, b)| > |f (a, b)| − |f (a, b)|/2 > 0. Contradição! Portanto

g(xk) −→ g(a) ⇒ g é contínua.

Exercício 4

Seja K ⊂ Rn compacto. Prove que a projeção π : Rm× Rn −→ Rm transforma todo subconjunto

fechado F ⊂ Rm× K num conjunto fechado π(F ) ⊂ Rm

. Dê exemplo de F ⊂ Rm× Rnfechado

(16)

Solução. Seja a ∈ π(F ). Então existe (xk = π(xk, yk))k∈N ∈ π(F ) tal que lim

k∈Nxk = a.

Como (xk, yk) ∈ F =⇒ yk ∈ K , logo como K é compacto ∃(yk)k∈N0 ⊂ (yk)k∈Ntal que lim

k∈N0

yk = b.

Logo lim

k∈N0

(xk, yk) = (a, b) ∈ F pois F é fechado. Então a = π(a, b) ∈ π(F ).

Assim temos que π(F ) ⊂ π(F ), e como sempre π(F ) ⊂ π(F ), logo π(F ) = π(F ) ⇔ π(F ) é fechado.

Exemplo:

Considere C = {(x, y) : x > 0, xy ≥ 1} ⊂ R2 um conjunto fechado. π : C −→ R tal que (x, y) 7−→ π(x, y) = x, ∀(x, y) ∈ C.

π(C) = (0, +∞) não é fechado.

1.1.8

Continuidade uniforme

Exercício 1

Sejam F , G ⊂ Rn fechados disjuntos não-vazios. A função contínua f : Rn −→ [0, 1], definida por f (x) = d(x, F )

d(x, F ) + d(x, G) cumpre f (x) = 0 para todo x ∈ F e f (x) = 1 para todo x ∈ G. Ela se chama a função de Urysohn do par (F, G). Prove que se ela é uniformemente contínua, então d(F, G) > 0.

Solução. Vamos assumir, por absurdo, que d(F, G) = 0. Então existem xk ∈ F e yk ∈ G com

|xk− yk| <

1

k (consequência da definição de distância).

Além disso para qualquer ε > 0, d(F, G) + ε > |x − y| para algum x ∈ F e y ∈ G. Dessa maneira, lim |xk − yk| = 0, mas observe que f (xk) = 0 e f (yk) = 1. Assim, |f (xk) − f (yk)| = 1 ⇒

lim |f (xk) − f (yk)| = 1 e consequentemente f não é uniformemente contínua.

Exercício 2

Seja Y ⊂ X ⊂ Rmcom Y denso em X. Se a aplicação contínua f : X −→ Rné tal que sua restrição f |Y é uniformemente contínua, prove que f é uniformemente contínua.

Solução. f |Y uniformemente contínua ⇒ dado ε > 0 arbitrário, ∃δ = δ(ε) > 0 tal que para

todo x e y em Y satisfazendo |x − y| < δ, tem-se |f (x) − f (y)| < ε/2. Tomemos então x0 e y0 em X tais que |x0 − y0| < δ. Por hipótese Y é denso em X , portanto existem sequências (x

k) e

(yk) em Y , tais que xk −→ x0 e yk −→ y0. Daí |x0 − y0| < δ ⇒ ∃ k0 ∈ N tal que ∀ k > k0

tem-se |xk− yk| < δ e portanto |f (xk) − f (yk)| < ε/2. Usando a continuidade de f concluimos que

|f (x0) − f (y0)| = lim |f (x

(17)

Exercício 3

Seja X ⊂ Rm um conjunto limitado. Se f : X → Rn é uniformemente contínua, prove que f (X) ⊂ Rn

também é limitado.

Solução. Se f (X) fosse ilimitada, para cada k ∈ N existiria xk ∈ X tal que |f (xk)| > k. A

sequência assim obtida não possuiria subsequência convergente.

Mas X é limitado, então existe N0 ⊂ N tal que (xk)k∈N0 é de Cauchy. Sendo f uniformemente

con-tínua, temos que (f (xk))k∈N0 é também de Cauchy, logo convergente. Contradição, pois (f (xk))k∈N

não admite subsequência convergente . Portanto f (X) é limitada.

Exercício 4

Sejam f, g : X → R uniformemente contínuas no conjunto X ⊂ Rm. Prove que a soma f + g : X −→ R é uniformemente contínua e o mesmo se dá com o produto f · g : X → R caso f e g sejam limitadas.

Solução. Sejam f, g : X ⊂ Rn −→ R uniformemente contínuas. Logo, ∀ (x

k), (yk) ⊂ X

sequências tais que |xk− yk| −→ 0 temos |f (xk) − f (yk)| −→ 0 e |g(xk) − g(yk)| −→ 0.

Defina φ : X ⊂ Rn −→ R, em que φ(x) = f(x) + g(x). Vamos mostrar que φ é uniformemente

contínua. De fato, sejam xk, yk ∈ X sequências tais que |xk− yk| −→ 0. Assim,

|φ(xk) − φ(yk)| = |f (xk) + g(xk) − f (yk) − g(yk)|

≤ |f (xk) − f (yk)| + |g(xk) − g(yk)|.

Como f e g são uniformemente contínuas, segue que

|φ(xk) − φ(yk)| −→ 0

∀xk, yk ∈ X tais que |xk− yk| −→ 0. Portanto, φ é uniformemente contínua.

Agora, defina ψ : X ⊂ Rm −→ R, ψ(x) = f(x)g(x). Temos que

|ψ(xk) − ψ(yk)| = |f (xk)g(xk) − f (yk)g(yk)|

= |f (xk)g(xk) − f (xk)g(yk) + f (xk)g(yk) − f (yk)g(yk)|

≤ |f (xk)||g(xk) − g(yk)| + |g(yk)||f (xk) − f (yk)|.

Se f e g são limitadas, isto é, existem Mf, Mg > 0 tais que |f (x)| < Mf, ∀x e |g(y)| < Mg, ∀y,

então

(18)

Portanto, ψ é uniformemente contínua se f e g são limitadas.

Exercício 5

Seja C ⊂ Rnconvexo. Se x ∈ Rne x ∈ C são tais que |x−x| = d(x, C), prove que hx − x, y − xi ≤ 0 para todo y ∈ C.

Solução. Suponha que exista y ∈ C tal que hx − x, y − xi > 0. Defina z = (y − x)t + x. Então |z − x|2 = |(y − x)t − (x − x)|2 = t2|y − x|2 − 2t hy − x, x − xi + |x − x|2 Daí, ∀ t ∈ I = (0, 1) ∩  0,2 hy − x, x − xi |y − x|2  6= ∅, temos que t2|y − x|2− 2t hy − x, x − xi < 0

⇒ |z − x| < |x − x| ⇒ z /∈ C. Absurdo, pois C é convexo.

Exercício 6

Dado C ⊂ Rnconvexo e fechado, seja f : Rn −→ C definida por f (x) = x, onde x , é o único ponto de C tal que |x − x| = d(x, C). Prove que |f (x) − f (y)| ≤ |x − y| para quaisquer x, y ∈ Rn, logo f é uniformemente contínua.

Solução. Sabemos pelo exercício anterior que se C é convexo , x ∈ Rn e x ∈ C são tais que

|x − x| = d(x, C), então hx − x, y − xi ≤ 0 para todo y ∈ C.

Como C é fechado ∃ x0, y0 ∈ C tal que para x, y ∈ Rntemos, |x−x0| = d(x, C) e |y −y0| = d(y, C).

Logo, pelo comentário inicial, temos

hx − x0, y0− x0i ≤ 0, hy − y0, x0− y0i ≤ 0 hy0− x0, x − x0i ≤ 0, hy0− x0, y0− yi ≤ 0 logo hy0 − x0, x − x0i + hy0− x0, y0− yi ≤ 0 hy0− x0, x − x0+ y0− yi ≤ 0 hy0− x0, (y0− x0) − (y − x)i ≤ 0 |y0− x0|2 ≤ hy0− x0, y − xi

Pela desigualdade de Schwarz

(19)

|y0− x0| ≤ |y − x|

Assim

|f (y) − f (x)| ≤ |y − x| então f é lipschitziana, portanto uniformemente contínua.

1.1.9

Homeomorfismos

Exercício 1

Chama-se semi-reta de origem 0 em Rna um conjunto do tipo σ = {tv; t ≥ 0, 0 6= v ∈ Rn}. Seja X ⊂ Rn− {0} um conjunto compacto que tem um (único) ponto em comum com cada semi-reta com

origem 0. Prove que X é homeomorfo à esfera Sn−1.

Solução. Seja ϕ : X −→ Sn−1a aplicação definida por ϕ(x) = x

|x|. Vamos mostrar que ϕ é um homeomorfismo.

Temos que ϕ é uma bijeção. De fato, dados x1, x2 ∈ X tais que ϕ(x1) = ϕ(x2), segue que

x1 |x1| = x2 |x2| ⇔ |x1| |x2|

x2 = x1 ⇔ x1 e x2 têm a mesma direção e o mesmo sentido, logo estão na

mesma semi-reta e assim x1 = x2, pois a interseção de cada semi-reta e o conjunto X é única. Logo,

ϕ é injetiva. Além disso, ∀ y ∈ Sn−1, ∃ t > 0 tal que ty ∈ X, pois y 6= 0, com ϕ(ty) = ty

|ty| = ty

t|y| = y

|y| = y. Dessa maneira, ϕ é também sobrejetiva. Temos ainda que ϕ é contínua, pois ϕ(x) = x

|x| é um quociente de funções contínuas (x ∈ X ⊂ Rn− {0} ⇒ |x| 6= 0).

Como X é compacto, logo ϕ é um homeomorfismo.

Exercício 2

Estabeleça um homeomorfismo entre Rn− {0} e o produto cartesiano Sn−1× R ⊂ Rn+1.

Solução. Defina f : Sn−1 × R −→ Rn − {0} pondo f (x, t) = etx. Temos que f é contínua

pois é o produto de funções contínuas. Além disso, g : Rn − {0} −→ Sn−1 × R, definida por

g(y) = y

|y|, ln |y| 

, é contínua e satifaz g(f (x, t)) = (x, t) e f (g(y)) = y. Portanto, f : Sn−1× R −→ Rn− {0} é um homeomorfismo.

Exercício 3

Mostre que existe um homeomorfismo do produto cartesiano Sm × Sn sobre um subconjunto de

(20)

Solução. Sm×Sn⊂ Sm ×Rn+1 ∼ Sm ×R×Rn ∼ (Rm+1 −{0})×Rn ⊂ Rm+n+1. Daí, olhando

para a função inclusão temos que Sm× Sné homeomorfo a um subconjunto de Sm× Rn+1( a saber,

o próprio Sm × Sn), mas este, por sua vez é homeomorfo a um subconjunto de Sm

× R × Rn, que

por sua vez é homeomorfo a um subconjunto de (Rm+1− {0}) × Rn ⊂ Rm+n+1, como queríamos.

Exercício 4

Dê exemplo de conjuntos X, Y ⊂ Rn e pontos a ∈ X, b ∈ Y tais que X − {a} e Y − {b} são

homeomorfos mas X não é homeomorfo a Y .

Solução. Sejam X = [0, 2π) o intervalo semi-aberto e Y = S1 = {(x, y) ∈ R2; x2 + y2 = 1} o

círculo unitário.

a) Mostraremos que a aplicação f : X − {a} −→ Y − {b}, onde a = 0 e b = (1, 0), definida por f (t) = (cos t, sen t) é um homeomorfismo. Com efeito, é claro que a aplicação f é contínua, além disso, f é bijeção. Mostrar que f−1 é contínua, é equivalente a mostrar que f (F ), donde F ⊂ (X − {a}), é um conjunto fechado. Com efeito, suponhamos que F ⊂ (0, 2π) é fechado (sa-bemos que F é limitado) então F é compacto, logo f (F ) é um conjunto compacto, o qual implica que f (F ) é fechado em S1− {(1, 0)}, portanto f−1é contínua, e concluímos que f é um

homeomor-fismo.

b) Agora mostraremos que a aplicação f : X −→ Y não é um homeomorfismo. Com efeito, é claro que a aplicação f definida por f = (cos t, sen t) é contínua e bijetiva. Mas a sua inversa f−1 : S1 → [0, 2π) é descontínua no ponto p = (1, 0). De fato, ∀k ∈ N, sejam tk = 2π −

1 k e zk = f (tk). Então lim

k→∞f (tk) = limk→∞zk= (1, 0), mas limk→∞f −1

(zk) = lim

k→∞tk = 2π 6= 0 = f −1

(1, 0), assim f−1é descontínua em (1, 0). Portanto f não é homeomorfismo.

Exercício 5

Sejam X ⊂ Rm, Y ⊂ Rn compactos, a ∈ X e b ∈ Y . Se X − {a} é homeomorfo a Y − {b}, prove que X e Y são homeomorfos.

Solução. Seja X − {a} ≈ X − {b}. Definaϕ g : X → Y x 7→    ϕ(x) se x 6= a b se x = a Note que g é bijetiva!

Para verificarmos que g é contínua, basta provarmos que lim

(21)

Ora, tomemos então (xk) ⊂ X − {a} tal que xk −→ a e suponha que g(xk) 6−→ b. Desse modo

devem existir  > 0 e N0 inf⊂ N tal que ϕ(xk) = g(xk) /∈ B(b, ), ∀ k ∈ N0.

Chamemos ϕ(xk) = yk. Então, como Y é compacto e (yk) ⊂ Y ⇒ ∃ N00 ⊂ N0 e c 6= b ∈ Y

tal que yk k∈N00

→ c. Mas ϕ é bijetiva ⇒ ∃ a ∈ X − {a} tal que ϕ(a) = c, e então usando o fato que ϕ é homeomorfismo, segue que xk = ϕ−1(yk)

k∈N00

→ ϕ−1(c) = a, onde a 6= a. Contradição! Portanto

g(xk) −→ b. Como X é compacto e g é bijetiva e contínua, segue que g é homeomorfismo de X

sobre g(X) = Y .

1.1.10

Conjuntos conexos

Exercício 1

Prove que um conjunto X ⊂ Rné conexo se, e somente se, para cada par de pontos a, b ∈ X existe

um conjunto conexo Cab ⊂ X tal que a ∈ Cabe b ∈ Cab.

Solução.

(⇒) Se X é conexo, basta tomar Cab = X sempre.

(⇐) Seja a ∈ X fixo. Então, para todo x ∈ X existe um conjunto conexo Cax ⊂ X tal que

a, x ∈ Cax. Logo, X =

[

x∈X

Cax. Como os conjuntos Cax são conexos e têm em comum o ponto a

entao X é conexo.

Exercício 2

Seja Z ⊂ Rn(n ≥ 2) um conjunto enumerável. Dados arbitrariamente os pontos a, b ∈ Rn− Z, prove que existe c ∈ Rntal que os segmentos de reta [a, c] e [c, b] estão ambos contidos em Rn− Z.

Conclua que o complementar de um conjunto enumerável em Rné conexo.

Solução. Considere em Rnuma reta r que intersepta o segmento [a, b] em seu ponto médio. Dados

x, y ∈ r onde x 6= y, os conjuntos [a, x] ∪ [x, b] = Ax e Ay = [a, y] ∪ [y, b] têm apenas os pontos

a, b em comum. Suponha por absurdo, que nenhum dos Ax, x ∈ r, estivesse contido em Rn− Z,

escolheríamos para cada x ∈ r um ponto f (x) ∈ Ax ∩ Z. Isto define uma aplicação f : r −→ Z

injetiva, a qual que não existe pois r é não enumerável e Z por hipotese é enumerável. Logo ∃c ∈ r tal que Ac= [a, c] ∪ [c, b] ⊂ Rn− Z. Daí podemos concluir que todo complementar de um conjunto

(22)

Exercício 3

Prove que S1 e S2não são homeomorfos. Solução. S1 − {p} ∼

= R e S2 − {q} ∼= R2, ambos através da projeção estereográfica. Daí

tomando p 6= p0temos que S1− {p, p0} ∼

= R − {P }, portanto S1− {p, p0} é desconexo.

Por outro lado, para q 6= q0 temos que S2 − {q, q0} ∼

= R2 − {Q}, portanto S2 − {q, q0} é conexo

por caminhos, logo conexo. Desse modo S1 não é homeomorfo a S2, pois se assim fosse teríamos S2− {q, q0} ∼

= S1− {p, p0}, o que não ocorre.

Exercício 4

Prove que S1 não é homeomorfo a subconjunto de R.

Solução. Um subconjunto de R, para ser homeomorfo a S1deveria ser compacto e conexo, logo

seria uma intervalo [a, b], o qual fica desconexo pela remoção de um ponto interior, mas a remoção de qualquer um dos seus pontos não desconecta S1.

Exercício 5

Quantas componentes conexas tem o conjunto X = {(x, y) ∈ R2; (x · y)2 = (x · y)}? Especifique-as.

Solução. X é a união dos dois eixos coordenados (onde ambos contem a origem) com os dois ramos da hipérbole. Portanto X tem três componentes conexas, onde a união dos dois eixos representa uma componente e cada um dos ramos da hipérbole é também uma componente conexa.

1.1.11

Limites

Exercício 1

Se f : X −→ Rné uniformemente contínua no conjunto X ⊂ Rm, prove que, para todo a, ponto de

acumulação de X, existe lim

x→af (x).

Solução. Sendo f é uniformemente contínua, toda sequência de Cauchy de pontos (xk) é levada

em uma sequência de Cauchy (f (xk)). Em particular, para toda sequência de pontos (xk) ∈ X − {a}

com lim xk = a existe lim f (xk) = b. Este valor não depende da sequência escolhida, pois se

tivéssemos outra sequência (yk) tal que lim yk = a e lim f (yk) = c 6= b, então definiríamos a

sequência (zk) ∈ X − {a} tal que zk = xk, se k é par e zk = yk, se k é ímpar. Neste caso a

sequência (zk) ainda cumpriria lim zk = a, mas lim f (zk) não existiria em virtude de (f (zk)) possuir

(23)

Exercício 2

Seja Y ⊂ X ⊂ Rm, com Y denso em X. Para toda aplicação uniformemente contínua f : Y −→ Rn, prove que existe uma única aplicação F : X −→ Rn, uniformemente contínua, tal que F (y) = f (y)

para todo y ∈ Y .

Solução. Como Y é denso em X e f é uniformemente contínua em Y , existe lim

y→xf (y) para todo

x ∈ X. Isto define F : X −→ Rn. Para todo ε > 0 dado, tome-se δ > 0 tal que y, y0 ∈ Y , |y −y0| < δ

⇒ |f (y) − f (y0)| < ε/2. Agora se x, x0 ∈ X e |x − x0| < δ, tomamos sequências (y

k) e (yk0) em Y ,

com lim yk = x e lim yk0 = x0. Desprezando alguns termos iniciais, podemos supor que |yk− yk0| < δ

onde |f (y) − f (y0)| < ε/2 para k ∈ N, logo |f (x) − f (x0)| = lim |f (yk) − f (yk0)| ≤ ε/2 < ε.

Exercício 3

Dada f : Rm −→ Rn, diz-se que se tem lim

x→∞f (x) = ∞ quando para todo B > 0 existe A > 0 tal

que |x| > A ⇒ |f (x)| > B. Se p : R2 −→ R2 é um polinômio complexo não-constante, prove que

lim

z→∞p(z) = ∞.

Solução. Seja p : R2 −→ R2, onde p(z) = a

0 + a1z + . . . + akzk, polinômio complexo não

constante. Temos que:

p(z) = zka0 zk + a1 zk−1 + · · · + ak−1 z + ak  . Tome ϕ(z) = a0 zk + a1 zk−1 + · · · + ak−1 z . Afirmação (*) lim z→∞ϕ(z) = 0 , isto é, dado c 2 = ε > 0, ∃δ > 0 tal que |z| > δ ⇒ |ϕ(z)| < c 2, onde c = |ak|. Logo |p(z)| = |zk(ϕ(z) + a k)| = |zkϕ(z) + zka k| ≥ |zk||a k| − |zk||ϕ(z)|, para |z| > δ ≥ |zk|c − |zk|c 2 = |zk|c − c 2  = |zk|c 2. Portanto,

(24)

|p(z)| ≥ |zk|c

2, para |z| > δ ⇒ lim

z→∞|p(z)| = ∞.

Prova da Afirmação (*) lim

z→∞ϕ(z) = 0 , isto é, dado ε > 0, ∃δ > 0 tal que |z| > δ ⇒ |ϕ(z)| < ε

|ϕ(z)| = a0 zn + a1 zn−1 + · · · + an−1 z ≤ a0 zn + · · · an−1 z ≤ a0 z + · · · + an−1 z ≤ L |z| + · · · + L |z| = nL |z| onde L = max{|ai|, i = 0, . . . , n − 1}. Logo, tomando δ = nL ε , temos |z| > nL ε ⇒ |ϕ(z)| < nL |z| = nL nL ε = ε. Portanto, |ϕ(z)| < ε. Exercício 4

Seja X = {x = (x1, · · · , xn) ∈ Rn; x1 · x2· · · xn 6= 0}. Defina f : X −→ R pondo

f (x) = sen (x1· x2· · · xn) x1· x2· · · xn

. Prove que lim

x→0f (x) = 1.

Solução. Sabemos da Análise Real que lim

t→0

sen (t)

t = 1. Daí, dado ε > 0, ∃ δ > 0 tal que ∀ t ∈ R, 0 < |t| < δ ⇒ sen (t) t − 1 < ε. Se tomarmos em R

n a norma do máximo e assumirmos

δ < 1, então para todo x ∈ X, 0 < |x| < δ, temos 0 < |xi| ≤ |x| < δ, ∀ i = 1, 2, . . . , n, daí

0 < |x1· x2· · · xn| < δn< δ ⇒ sen (x1· x2· · · xn) x1· x2· · · xn − 1 < ε, como queríamos.

(25)

1.2 - Caminhos em R

n

1.2.1

Caminhos diferenciáveis

Exercício 1

Seja f : I −→ Rn um caminho diferenciável. Se existirem a ∈ I e b ∈ Rn tais que a é ponto de

acumulação do conjunto f−1(b), prove que f0(a) = 0.

Solução. Existe xk ∈ I − {a}, tal que lim xk = a e (xk) ⊂ f−1(b), ou seja, f (xk) = b, ∀ k > 0.

Mas f é contínua, logo f (a) = lim f (xk) = lim b = b. Então, por f ser diferenciável, f0(a) existe e

é unica, daí f0(a) = lim x→a f (x) − f (a) x − a = limk→∞ f (xk) − f (a) xk− a = lim k→∞ f (a) − f (a) xk− a = lim x→∞ 0 xk− a = 0. Exercício 2

Seja f : I −→ R2 um caminho diferenciável, cuja imagem coincide com o gráfico da função g :

[0, 1] −→ R e g(t) = |t|. Se a é um ponto interior de I tal que f (a) = (0, 0), prove que f0(a) = 0. Solução. Como a imagem de f coincide com o gráfico de g temos que f (t) = (x(t), |x(t)|), ∀ t ∈ I, com f (a) = (0, 0). Note que |x(t)| ≥ 0, ∀ t ∈ I ⇒ a é ponto de mínimo da função t 7→ |x(t)| e então a derivada desta função é zero em t = a . Assim, como

−|x(t)| ≤ x(t) ≤ |x(t)| ⇒ x0(a) = 0.

Portanto,

f0(a) = (x0(a), |x|0(a)) = (0, 0).

Exercício 3

Seja f : R −→ R3 a hélice cilíndrica, definida por f (t) = (cos t, sen t, t). Prove que, para todo

(26)

Solução. Temos f0(t) = (−sen t, cos t, 1) e f00(t) = (− cos t, −sen t, 0), então f (t) + f00(t) = (cost, sent, t) + (−cost, −sent, 0) = (0, 0, t) o qual já pertence ao eixo vertical de R3.

Exercício 4

O caminho g : R −→ R3, definido por g(t) = (a cos bt, asen bt, ct) , é também chamado de hélice. Determine a relação entre as constantes a, b, c a fim de que o caminho g esteja parametrizado pelo comprimento do arco.

Solução. Uma curva α é parametrizada pelo comprimento do arco se |α0(t)| = 1. Seja g : R −→ R3, definido por g(t) = (a cos bt, asen bt, ct).

Temos que g0(t) = (−absen bt, ab cos bt, c) logo, |g0(t)| = phg0(t), g0(t)i

= p(−absen bt)2+ (ab cos bt)2+ c2

= √a2b2sen2bt + a2b2cos2bt + c2

= pa2b2(sen2bt + cos2bt) + c2

= √a2b2+ c2.

Como queremos que g seja parametrizada pelo comprimento do arco temos que ter |g0(t)| = 1

⇒ √a2b2+ c2 = 1

⇒ a2b2+ c2 = 1.

1.2.2

Cálculo diferencial de caminhos

Exercício 1

Seja f : [a, b] −→ Rnum caminho diferenciável tal que f (a) = f (b) = 0. Prove que existe c ∈ (a, b) tal que hf (c), f0(c)i = 0.

Solução. Seja g : [a, b] −→ R; g(t) = hf (t), f (t)i. Temos que g é contínua em [a, b] e dife-renciável em (a, b). Além disso g(a) = g(b). Daí, pelo teorema do valor médio para funções reais, temos que existe c ∈ (a, b) tal que 0 = g(b) − g(a) = g0(c)(b − a) = 2 hf (c), f0(c)i (b − a) ⇒ hf (c), f0(c)i = 0.

(27)

Exercício 3

Seja f : I −→ Rn2 um caminho diferenciável cujos valores são matrizes n × n. Prove que g : I −→ Rn2

, dado por g(t) = f (t)k, é diferenciável e calcule g0(t).

Solução. Temos que g é diferenciável, pois é a composta t 7→ f (t) 7→ (f (t), · · · , f (t)) → f (t)ϕ k,

onde ϕ : Rn2

× · · · × Rn2

é a aplicação k-linear dada pelo produto de matrizes. A derivada da função f : R n2 −→ Rn2 x 7−→ xk é a transformação linear f0(x) : Rn2 → Rn2 , dada por f0(x) · v = k X i=1 xi−1· v · xk−i .

Em dimensão 1 e pela regra da cadeia

f0(t) = k X i=1 x(t)i−1· x(t)k−i ! · f0(t).

1.2.3

A integral de um caminho

Exercício 1

Sejam f : [a, b] −→ Rne ϕ : [a, b] → R de classe C1. Se |f0(t)| ≤ ϕ0(t) para todo t ∈ (a, b), prove

que |f (b) − f (a)| ≤ ϕ(b) − ϕ(a).

Solução. Pelo Teorema Fundamental Cálculo para caminhos temos: Z b a f0(t)dt = f (b) − f (a) ⇒ |f (b) − f (a)| = Z b a f0(t)dt ≤ Z b a |f0(t)|dt ≤ Z b a ϕ0(t)dt = ϕ(b) − ϕ(a) ∴ |f (b) − f (a)| ≤ ϕ(b) − ϕ(a). Exercício 2

Seja f : [a, a + h] −→ Rnum caminho de classe Ck. Prove que f (a + h) = f (a) + h · f0(a) + · · · + h k−1 (k − 1)!f k−1(a) + r k onde rk = hk (k − 1)! Z 1 0 (1 − t)k−1f(k)(a + th)dt.

(28)

Solução. Como f (t) = f1(t), . . . , fn(t)



é um caminho de classe Ck, segue que fi(t) : [a, a + h] −→ R é um caminho de classe Ck, ∀i = 1 . . . n. Pela fórmula de Taylor com

resto integral para funções reais temos que

fi(a + h) = fi(a) + h · fi0(a) + · · · + hk−1 (k − 1)!f k−1 i (a) + r i k, onde ri k = hk (k − 1)! Z 1 0 (1 − t)k−1fi(k)(a + th)dt, ∀ i = 1, . . . , n. Assim, f (a + h) = f1(a + h), . . . , fn(a + h)  = f1(a) + h · f10(a) + · · · + hk−1 (k − 1)!f k−1

1 (a) + r1k, . . . , fn(a) + h · fn0(a) + · · · +

+ h k−1 (k − 1)!f k−1 n (a) + rkn  = f (a) + h · f0(a) + · · · + h k−1 (k − 1)!f k−1(a) + r k, onde rk = (rk1, · · · , r n k) =  hk (k − 1)! Z 1 0 (1 − t)k−1f1(k)(a + th)dt, . . . , h k (k − 1)! Z 1 0 (1 − t)k−1fn(k)(a + th)dt  = h k (k − 1)! Z 1 0 (1 − t)k−1f(k)(a + th)dt. Exercício 3

Sejam f, g : [a, b] −→ Rncaminhos de classe C1. Prove que Z b

a

D

f (t), g0(t)Edt = hf (b), g(b)i − hf (a), g(a)i − Z b a D f0(t), g(t)Edt. Solução. Denotando f = (f1, . . . , fn) e g 0 = (g10, . . . , g0n), temos que Z b a D f (t), g0(t)Edt = Z b a f1(t)g 0 1(t) + · · · + fn(t)g 0 m(t)dt = Z b a f1(t)g10(t)dt + Z b a f2(t)g02(t)dt + · · · + Z b a fn(t)gn0(t)dt (∗) = f1(t)g1(t)|ba− Z b a f10(t)g1(t)dt + · · · + fn(t)gn(t)|ba− Z b a fn0(t)gn(t)dt = f1(b)g1(b) − f1(a)g1(a) − Z b a f10(t)g1(t)dt + · · · + fn(b)gn(b) − fn(a)gn(a)− − Z b a fn0(t)gn(t)dt

= hf (b), g(b)i − hf (a), g(a)i − Z b

a

D

(29)

(∗) Teorema da Integração por partes:

Se f, g : [a, b] −→ R possuem derivadas integráveis então Z b a f (t)g0(t)dt = f (t)g(t)|ba− Z b a f0(t)g(t)dt.

1.2.4

Caminhos retificáveis

Exercício 1

Seja f : [a, b] −→ Rn um caminho retificável, com f (a) = A e f (b) = B. Se seu comprimento é

l(f ) = |B − A|, prove que f é uma reparametrização do caminho retilíneo [A, B].

Solução. Para toda partição P = {a = t0 < t1 < · · · < tk = b} temos que

|B − A| ≤ l(f, P ) ≤ l(f ). Como l(f ) = |B − A|, segue-se que l(f, P ) = |B − A|. Resulta que os pontos A = f (t0), f (t1) · · · , f (tk) = B estão dispostos ordenadamente sobre o segmento de

reta AB. Então, ∀t ∈ [a, b], tem-se f (t) = A + ϕ(t) · v, com v = B − A, e a função ϕ : [a, b] −→ [0, b] é não-decrescente. Com f ∈ C1, segue-se que ϕ ∈ C1, como é não-decrescente, ϕ0 ≥ 0. Logo f é

uma reparametrização do caminho retilíneo f (t) = A + t · v.

Exercício 3

Seja U ⊂ Rn aberto e conexo. Dados a, b ∈ U , prove que existe um caminho retificável f : I → U

começando em a e terminando em b.

Solução. Seja a, b ∈ U . Como U é aberto e conexo, segue que U é conexo por caminhos, logo existe um caminho poligonal contido en U que liga a e b. Isto é, existem x0, x2, . . . , xn ∈ U tais

que o caminho retilíneo Pi : [0, 1] −→ U com Pi(0) = xi−1 e Pi(1) = xi está contido em U ,

∀ i = 1, . . . , n, onde x0 = a e xn = b. Defina o caminho f : [0, 1] → U como a justaposição dos

caminhos P1, P2, . . . , Pnpara uma partição P = {t0 < t1 < . . . < tn}. Assim,

l(f ; P ) = k X i=1 |f (ti) − f (ti−1)| = k X i=1 |xi− xi−1| ≤ nK, onde K = max

i=1,n{|xi− xi−1|}. Então l(f ; P ) é limitado para toda partição P . Portanto f é retificável.

Exercício 4

Dado U ⊂ Rnaberto e conexo, defina a distância intrínseca entre os pontos a, b ∈ U como o ínfimo

dU(a, b) dos comprimentos dos caminhos retificáveis f : I −→ U , que ligam a e b. Prove que se (xk)

(30)

Solução.

(⇐) Da definição de distância intrínseca entre os pontos x e a concluimos que |x − a| ≤ dU(x, a),

logo se lim dU(xk, a) = 0 ⇒ lim xk = a.

(⇒) Seja B = B(a; r) ⊂ U . Para pontos xk ∈ B, tem-se que dU(xk, a) = |x − a|, portanto

lim xk = a ⇒ lim |xk− a| = 0 ⇒ lim dU(xk, a) = 0, pois xk ∈ B para todo k suficientemente

grande.

1.3 - Funções Reais de n Variáveis

1.3.1

Derivadas parciais

Exercício 1

Um conjunto X ⊂ Rn chama-se i-convexo (1 ≤ i ≤ n) quando para quaisquer a, b ∈ X tais que b = a + tei, tem-se [a, b] ⊂ X. (Se X ⊂ R2, diz-se então que X é horizontalmente convexo ou

verticalmente convexo, conforme seja i = 1 ou i = 2). Prove que se o aberto U ⊂ Rné i-convexo e a função f : U −→ R cumpre ∂f

∂xi

(x) = 0 para todo x ∈ U então f não depende da i-ésima variável, isto é, x, x + tei ∈ U ⇒ f (x + tei) = f (x).

Solução. Como U é i-convexo, o segmento de extremos x e x + tei está contido em U .

Além disso, a existência de ∂f ∂xi

(x) = 0 para todo x ∈ U nos assegura que f é contínua em [x, x + tei] e é diferenciável em (x, x + tei), daí pelo Teorema do Valor Médio, ∃ θ ∈ (0, 1) tal

que f (x + tei) − f (x) =

∂f ∂xi

(x + θtei)t = 0 ⇒ f (x + tei) = f (x), como queríamos.

Exercício 2

Sejam X = {(x, 0); x ≥ 0} e U = R2 − X. Defina f : U −→ R pondo f(x, y) = x2 quando

x > 0, y > 0 e f (x, y) = 0 quando x ≤ 0 ou y < 0. Mostre que se tem ∂f

∂y = 0 em todos os pontos de U mas f depende de y.

Solução. O conjunto aberto U = R2 − X é horizontalmente convexo. E, para determinarmos as

derivadas parciais de f em relação à y, consideremos as duas restrições que definem f : (i) Para x > 0, y > 0, f (x, y) = x2 ⇒ ∂f

∂y = 0; (ii) Para x ≤ 0 ou y < 0, f (x, y) = 0 ⇒ ∂f

(31)

Para mostrar que f depende de y, basta mostrar que f assume valores diferentes para diferentes valores de y. Para tal, considerando x > 0, y > 0, temos que f (x, y) = x2 é estritamente positiva e,

tomando o simétrico da segunda coordenada (essa passa a ser negativa), obtemos f (x, −y) = 0.

Exercício 3

Diz-se que um caminho retilíneo f : I −→ Rn é paralelo ao i-ésimo eixo quando ele é da forma f (t) = a + tei, t ∈ I. Se U ⊂ Rn é um aberto conexo, prove que dois pontos a, b ∈ U quaisquer

podem ser ligados por um caminho poligonal contido em U , cujos trechos retilíneos são paralelos aos eixos. Conclua que se U ⊂ Rn é conexo e f : U −→ R cumpre ∂f

∂xi

(x) = 0 para todo x ∈ U e qualquer i com 1 ≤ i ≤ n, então f é constante.

Solução. Dois pontos quaisquer de uma bola podem ser ligados por um caminho poligonal contido nela, o qual tem seus lados paralelos aos eixos. Segue-se daí, que o mesmo ocorre em qualquer aberto conexo. Fixando a ∈ U , para todo ponto x ∈ U , unindo-o ao ponto a por um caminho desse tipo, em cada segmento retilíneo do caminho varia apenas a i-ésima coordenada, e como ∂f

∂xi

= 0, a função f se mantém constante ao longo desse segmento. Então f (x) = f (a) para todo x ∈ U e f é constante.

Exercício 4

Seja U ⊂ Rn aberto. Se f : U −→ R possui derivadas parciais ∂x∂fi : U −→ R, i = 1, . . . , n limitadas, prove que f é contínua.

Solução. Seja M > 0 tal que ∂f ∂xi (x) ≤ M , ∀ i = 1, . . . , n e ∀ x ∈ M .

Dados x, x + v ∈ U com v = (α1, . . . , αn) definamos a seguinte sequência de vetores em Rn:

v0 = 0 v1 = v0+ α1e1 v2 = v1+ α2e2 .. . ... ... vn = vn−1+ αnen= v. Daí f (x + v) − f (x) = f (x + v1) − f (x + v0) + f (x + v2) − f (x + v1) + . . . + f (x + vn) − f (x) ⇒ f (x + v) − f (x) = n X i=1 [f (x + vi) − f (x + vi−1)]

(32)

|f (x + vi) − f (x + vi−1)| = ∂f ∂xi (z) .|αi| ≤ M |αi|, z ∈ [vi−1, vi] Então, |f (x + v) − f (x)| ≤ M n X i=1

|αi| = M |v|, daí fazendo y = x + v obtemos que |f (y) − f (x)| ≤

M |x − y|, logo f é contínua.

1.3.2

Funções de classe C

1

Exercício 1

Seja f : R2 −→ R definida por f(x, y) = x

2y

x2 + y2 se (x, y) 6= (0, 0) e f (0, 0) = 0. Mostre que, para

todo v = (α, β) ∈ R2, existe a derivada direcional ∂f

∂v(0, 0) mas f não é diferenciável no ponto (0, 0).

Solução. Se v = (α, β) então ∂f ∂v(0, 0) = limt→0 f (tα, tβ) − f (0, 0) t = limt→0 (tα)2 (tα)2+ (tβ)2 1 t = α2β α2+ β2, ∀ v 6= 0. Em particular, ∂f ∂x(0, 0) = 0 e ∂f ∂y(0, 0) = 0 =⇒ ∇f (0, 0) =  ∂f ∂x, ∂f ∂y  (0, 0) = 0.

Se f fosse diferenciável no ponto (0, 0), teríamos ∂f

∂v(0, 0) = h∇(0, 0), vi, o que não ocorre. Exercício 2

Seja f : Rn−→ R uma função contínua que possui todas as derivadas direcionais em qualquer ponto de Rn. Se ∂f

∂u(u) > 0 para todo u ∈ S

n−1

, prove que existe a ∈ Rntal que ∂f

∂v(a) = 0, seja qual for v ∈ Rn.

Solução. Seja u ∈ Sn−1. Então a condição ∂f

∂u(u) > 0 implica que existe δ > 0 tal que para todo t ∈ R satisfazendo −δ < t < 0 tem-se f (u + tu) − f (u)

t > 0 ⇒ f (u + tu) < f (u).

Note que se −δ < t < 0 então 1 − δ < 1 + t < 1 ⇒ |(1 + t)u| < |u| = 1 e assim (1 + t)u ∈ B(0, 1). Além disso, f (1 + t)u < f (u). Como esta desigualdade vale para todo u ∈ Sn−1, temos que o

mínimo de f |B[0,1] é assumido em algum ponto a ∈ B(0, 1).

Definindo ϕv : R −→ R por ϕv(t) = f (a + tv), ∀ v ∈ Rn, temos que ϕ tem um mínimo local quando

t = 0 e assim 0 = ϕ0v(0) = ∂f ∂v(a).

(33)

Outra Solução.

Temos que ∂f

∂u(u) = limt−→0

f (u + tu) − f (u) t > 0. Considere ϕ : R −→ R definida por ϕu(t) = f (tu).

Como ϕ0u(1) = lim h−→0 ϕu(1 + h) − ϕu(1) h = limh−→0 f (1 + h)u − f (u) h = ∂f ∂u(u) > 0 ⇒ ϕ 0 u(1) > 0,

então existe ε > 0 tal que 1 − ε < t < 1 ⇒ ϕu(t) < ϕu(1).

Assim,

f (tu) < f (u), 1 − ε < t < 1, u ∈ Sn−1. (i)

Como f é contínua na bola fechada B[0, 1], pelo Teorema de Weierstrass, f assume um mínimo nesse conjunto, o qual é atingido num ponto a tal que |a| < 1. Se essa desigualdade não fosse estrita, teríamos que a ∈ Sn−1e assim, de (i), a não seria ponto de mínimo.

Como a ∈ intB[0, 1], temos que a + tv ∈ B[0, 1], para t suficientemente pequeno. Definindo ψ : R −→ R por ψv(t) = f (a + tv), segue que

ψv(t) = f (a + tv) ≥ f (a) = ψv(0), para cada v ∈ Rn. (ii)

Logo, ∂f ∂v(a) = limt−→0 f (a + tv) − f (a) t = limt−→0 ψv(t) − ψv(0) t = ψ 0 v(0) = 0, ∀ v ∈ R n,

pois, por (ii), 0 é um ponto de mínimo local de ψ para cada v ∈ Rn.

Exercício 3

Seja f : Rn → R diferenciável no ponto 0. Se f(tx) = t · f(x) para todo t > 0 e todo x ∈ Rn,

prove que f é linear. Conclua que a função ϕ : R2 −→ R, dada por ϕ(x, y) = x3/(x2 + y2) se

(x, y) 6= (0, 0) e ϕ(0, 0) = 0, não é diferenciável na origem.

Solução. Primeiro note que f diferenciável em 0 ⇒ f contínua em 0. Como ∀ t > 0, f (tx) = tf (x), então lim t→0+f (tx) = f  lim t→0+tx  = f (0). Além disso, f (tx) = tf (x) ⇒ f (0) = lim t→0+f (tx) = limt→0+tf (x) = 0.

Por outro lado, temos que f0(0)x = lim t→0 f (tx) − f (0) t = limt→0+ f (tx) − f (0) t = limt→0+f (x) = f (x), ∀ x ∈ R n. Portanto f é linear.

(34)

No caso da função ϕ : R2 −→ R dada por ϕ(x, y) = x

3

x2+ y2 se x

2+ y2 6= 0 e ϕ(x, y) = 0 se x2+ y2 = 0,

temos que f (tx, ty) = t

3x3 t2(x2+ y2). Daí, ∀ t > 0, ϕ(tx, ty) = t  x3 x2+ y2  = tϕ(x, y) e então se ϕ fosse diferenciável em (0, 0), pelo que foi provado anteriormente, teríamos ϕ : R2 −→ R linear, o

que não ocorre.

Exercício 4

Seja f : U → R de classe C1 no aberto U ⊂ Rn. Prove que, dados a ∈ U e ε > 0, ∃ δ > 0 tal que x, y ∈ U, |x−a| < δ, |y −a| < δ ⇒ f (x)−f (y) = h∇f (a), x−yi+r(x, y), onde |r(x, y)| < ε|x−y|.

Solução. f ∈ C1 ⇒ r(x) = f (x) − f (a) − f0(a)(x − a) ∈ C1(U ) e ∂r

∂xi

(a) = 0, i = 1, . . . , n, então dado ε > 0, existe δ > 0 tal que

∂r ∂xi (x) − ∂r∂xi(a) < ε, ∀ x com |x − a| < δ.

Então pelo T.V.M., |x − a| < δ, |y − a| < δ ⇒ |r(x) − r(a)| < ε|x − y|, pois B(a; δ) é convexa. Agora note que f (x) = f (a) + f0(a)(x − a) + r(x) e f (y) = f (a) + f0(a)(y − a) + r(y) implicam f (x) − f (y) = f0(a)(x − y) + r(x) − r(y).

r(x, y) := r(x) − r(y) ⇒ f (x) − f (y) = h∇f (a), x − yh + r(x, y), onde |r(x, y)| < ε|x − y|.

1.3.3

O Teorema de Schwarz

Exercício 1

Seja f : I × J −→ R de classe C2no retângulo aberto I × J ⊂ R2. Se ∂ 2f

∂x∂y é identicamente nula, prove que existem ϕ : I −→ R, ψ : J −→ R de classe C2 tais que f (x, y) = ϕ(x) + ψ(y) para todo (x, y) ∈ I × J .

Solução. Como ∂

2f

∂x∂y e ∂2f

∂y∂x são identicamente nulas, ∂f

∂y não depende de x e ∂f

∂x não depende de y. Fixando (x0, y0) ∈ I × J podemos então definir as funções ϕ : I −→ R e

ψ : J −→ R pondo ϕ(x) = ∂f

∂x(x, y0) e ψ(y) = ∂f

∂y(x0, y), as quais são de classe C

1 e

cum-prem ϕ(x) = ∂f

∂x(x, y), ψ(y) = ∂f

(35)

f (x, y) = f (x, y) − f (x0, y) + f (x0, y) − f (x0, y0) + f (x0, y0) = Z x x0 ∂f ∂x(s, y)ds + Z y y0 ∂f ∂y(x0, t)dt + f (x0, y0) = Z x x0 ϕ(s)ds + Z y y0 ψ(t)dt + f (x0, y0) = ϕ(x) + ψ(y). Exercício 2

Use o exercício anterior para provar que se g : R × R → R é de classe C2, com ∂ 2g

∂x2 =

∂2g ∂y2, então

existem ϕ : R −→ R e ψ : R −→ R de classe C2, tais que g(x, y) = ϕ(x + y) + φ(x − y) para todo (x, y).

Solução. Definamos f : R2 −→ R, pondo f(u, v) = g (u + v, u − v). ∂f ∂u = ∂g ∂x ∂x ∂u + ∂g ∂y ∂y ∂u =  ∂g ∂x + ∂g ∂y  e ∂2f ∂v∂u = ∂2g ∂x2 ∂x ∂v + ∂2g ∂y∂x ∂y ∂v + ∂2g ∂x∂y ∂x ∂v + ∂2g ∂y2 ∂y ∂v =  ∂ 2g ∂x2 − ∂2g ∂y2  +  ∂2g ∂x∂y − ∂2g ∂y∂x  =  ∂ 2g ∂y2 − ∂2g ∂x2  ⇒ ∂ 2f ∂v∂u = 0.

Logo f satisfaz as condições do exercício anterior, donde segue que existem ϕ : R −→ R e ψ : R −→ R tais que f (u, v) = ϕ(u) + ψ(v) = g (u + v, u − v), fazendo u + v = x e u − v = y temos u = x + y e v = x − y

(36)

Exercício 3

Seja f : Rn −→ R de clase C2, tal que f (t, x) = t2

f (x) para todo t > 0 e todo x ∈ Rn. Prove que existem aij ∈ R (i, j = 1, . . . , n) tais que f(x) =

n

P

i,j=1

ai,jxixj para todo x = (x1, . . . , xn) ∈ Rn.

Como explicar f (x, y) = x

4+ y4

x2+ y2?

Solução.

(i) Derivando a igualdade t2f (x) = f (tx) em relação a t, obtemos 2tf (x) =

n P j=1 ∂f ∂xj (t, x)xj onde

se usou a regra da cadeia. Derivando outra vez em relação a t (isso é possível, pois f ∈ C2): 2f (x) = n X i=1 n X j=1 ∂2f ∂xi∂xj (tx)xixj, ou seja, f (x) = 1 2 n X i=1 n X j=1 ∂2f ∂xi∂xj (tx)xixj.

Tomando o limite quando t −→ 0, obtemos f (x) = 1 2 n X i=1 n X j=1 ∂2f ∂xi∂xj  lim t→0tx  xixj = 1 2 n X i=1 n X j=1 ∂2f ∂xi∂xj (0)xixj = 1 2 n X i=1 n X j=1 aijxixj, onde aij = ∂2f ∂xi∂xj (0). (ii) f (x, y) = x 4+ y4 x2+ y2 não é de classe C

2, pois não tem derivadas parciais contínuas no ponto

(0, 0). Portanto, f (x, y) 6= 2 P i,j=1 aijxy. Exercício 4

Sejam f, ϕ : U −→ Rn+1 de classe C2 no aberto U ⊂ Rn. ( Isto é, as funções-coordenada de f e ϕ são de classe C2.) Suponha que



f (x), ∂ϕ ∂xi

(x) 

= 0 para todo x ∈ U e todo i = 1, . . . n. Prove que a matriz [aij(x)], onde aij(x) =  ∂f ∂xi (x), ∂ϕ ∂xj (x) 

, é simétrica, seja qual for x ∈ U .

Solução. Temos que f, ϕ : U −→ Rn+1são de classe C2 . Seja

 f (x), ∂ϕ ∂xi (x)  = 0 ∀x ∈ U e todo i = 1, . . . , n. Em particular,  f (x), ∂ϕ ∂xj (x)  = 0.

Logo, derivando a primeira igualdade em relação a xj temos:

 ∂f ∂xj (x), ∂ϕ ∂xi (x)  +  f (x), ∂ 2ϕ ∂xjxi (x)  = 0 (1)

(37)

e derivando a segunda igualdade em relação a xi temos:  ∂f ∂xi (x), ∂ϕ ∂xj (x)  +  f (x), ∂ 2f ∂xixj (x)  = 0. (2) Igualando (1) e (2) obtemos:  ∂f ∂xj (x), ∂ϕ ∂xi (x)  +  f (x), ∂ 2ϕ ∂xjxi (x)  = ∂f ∂xi (x), ∂ϕ ∂xj (x)  +  f (x), ∂ 2f ∂xixj (x)  ⇒  ∂f ∂xj (x), ∂ϕ ∂xi (x)  +  f (x), ∂ 2ϕ ∂xjxi (x)  −  f (x), ∂ 2f ∂xixj (x)  = ∂f ∂xi (x), ∂ϕ ∂xj (x)  .

Pelo Teorema de Schwarz, segue que  ∂f ∂xj (x), ∂ϕ ∂xi (x)  = ∂f ∂xi (x), ∂ϕ ∂xj (x)  . Portanto a matriz [aij], onde aij(x) =

 ∂f ∂xi (x), ∂ϕ ∂xj (x)  é simétrica.

1.3.4

A fórmula de Taylor

Exercício 1

Seja r : U −→ R uma função de classe Ck definida num aberto U ⊂ Rnque contém a origem 0. Se r, juntamente com todas as suas derivadas parciais até as de ordem k, se anulam no ponto 0, prove que lim

v→0

r(v) |v|k = 0.

Solução. Provaremos por indução sobre k.

Para k = 1 a afirmação é verdadeira, pois por hipótese r é diferenciável e r0(0) = r(0) = 0, então r(v) = r(0) + r0(0)v + r(v), onde 0 = lim v→0 r(v) |v| = limv→0 r(v) |v| .

Supondo o resultado válido para k − 1 e seja r uma função k vezes diferenciável em 0, com todas as derivadas parciais de ordem menor ou igual a k nulas na origem. Então para cada i = 1, . . . , n a função ∂r

∂xi : U → R é k − 1 vezes diferenciável e também tem todas as derivadas parciais de ordem

menor ou igual a k − 1 nulas na origem.

Daí, pela hipótese de indução, temos que lim

v→0 ∂r ∂xi(v)

|v|k−1 = 0. Pelo Teorema do Valor Médio, existe

θ ∈ (0, 1) tal que r(v) − r(0) = n X i=1 ∂r ∂xi (θv)vi, onde r(0) = 0. Daí, r(v) |v|k = n X i=1 ∂r ∂xi(θv) |v|k−1 · vi |v|. Note que, para todo i = 1, . . . , n, vi

|v| é limitado, então no limite temos limv→0

r(v) |v|k = 0.

(38)

1.3.5

Pontos críticos

Exercício 1

Uma função f : U −→ R, de classe C2 no aberto U ⊂ Rn, chama-se harmônica quando n

X

i=1

∂2f ∂xi∂xi

(x) = 0 para todo x ∈ U . Prove que a matriz hessiana de uma função harmônica não pode ser definida (nem positiva nem negativa).

Solução. Se [hij] é a matriz da forma quadrática H então hii = H · v2, com v = ei =

(0, . . . , 1, . . . , 0). Portanto os elementos da diagonal da matriz de uma forma quadrática positiva (ou negativa) são todos números positivos (ou negativos) e assim sua soma não pode ser igual a zero.

Exercício 2

Sejam f : U −→ R uma função arbitrária,definida num aberto U ⊂ Rn. Prove que o conjunto dos

pontos de máximo (ou de mínimo) local estrito de f é enumerável.

Solução. Seja U o conjunto dos pontos de máximo local estrito de f . Dado x ∈ U, ∃ B(x; 2δ) ⊂ U , tal que y ∈ B(x; 2δ), y 6= x ⇒ f (y) < f (x) ( pois U é aberto e x é máximo local estrito). Para cada x ∈ U escolhamos um ponto qx ∈ Qn∩ B(x; δ), (isto é possível pois Qné denso em Rn) e um

número racional rx > 0 tal que |x − qx| < rx< δ , portanto B(qx; rx) ⊂ B(x; 2δ) e daí y ∈ B(qx, rx)

com y 6= x ⇒ f (y) < f (x).

A correspondência x 7→ (qx, rx) é injetiva pois se qx = qx0 e rx = r

x0 então x

0

∈ B(qx; rx) e

x ∈ B(qx0; rx0). Se fosse x 6= x0, teríamos f (x0) < f (x) e f (x) < f (x0), o que é um absurdo.

Disto segue que f é injetiva e assim existe uma correspondência injetiva entre os elementos de U e um subconjunto de Q × Q, portanto U é enumerável.

Exercício 3

Determine os pontos críticos de função f : R2 −→ R, f(x, y) = cos(x2 + y2) e da função

g(x, y) = x3− y3− x + y. Solução. Como ∇f (x, y) =  ∂f ∂x, ∂f ∂y 

= −2sen (x2+ y2) · (x, y), os pontos críticos de f são aqueles para os quais ∇f (x, y) = (0, 0). Temos x = y = 0 ou sen(x2 + y2) = 0, i.é, a origem e os

pontos (x, y) com x2+ y2 = kπ, k ∈ N (circunferências com centro na origem e raio√kπ).

(39)

−3y2+ 1 = 0, assim os pontos críticos são A = √ 3 3 , √ 3 3 ! , B = − √ 3 3 , √ 3 3 ! , C = √ 3 3 , − √ 3 3 ! e D = − √ 3 3 , − √ 3 3 ! .

A matriz Hessiana Hg(x, y) é dada por  

6x 0

0 −6y 

. Vejamos a natureza dos pontos críticos. Seja v = (α, β). No ponto A, tem-se ( α β )   2√3 0 0 −2√3     α β  = 2 √ 3(α2− β2 ).

Logo a forma quadrática é indefinida. Portanto, A é um ponto de sela.

Analogamente podemos observar que C é un ponto mínimo local, B é um máximo local e D é outro ponto de sela.

Exercício 4

Seja f : U −→ R diferenciável no aberto limitado U ⊂ Rn. Se, para todo a ∈ ∂U , tem-se

lim

x→af (x) = 0, prove que existe em U pelo menos um ponto crítico de f .

Solução. Defina F : ¯U ⊂ Rn−→ R por F (x) = f(x), se x ∈ U, e F (x) = 0, se x ∈ ∂U.

Por hipótese, temos que f |U é contínua, pois f é diferenciável em U .

Além disso, se a ∈ ∂U , então lim

x→aF (x) = limx→af (x) = 0 hip.

= F (a). Logo, F é contínua em ¯U . Como ¯U é compacto, pelo Teorema de Weierstrass, f assume máximo e mínimo em ¯U . Como F (x) = 0, ∀ x ∈ ∂U , então, exceto se F for identicamente nula, pelo menos um ponto crítico (máximo ou mínimo) é assumido em U . Portanto, f possui pelo menos um ponto crítico.

Exercício 5

Determine os pontos críticos da função f : R2 −→ R dada por f(x, y) = x2+ y2+ (x2− y2− 1)2 e

calcule as matrizes hessianas correspondentes. Solução. Temos que ∂f

∂x(x, y) = 2x + 2(x

2 − y2 − 1)2x e ∂f

∂y(x, y) = 2y − 2(x

2 − y2 − 1)2y.

Então os pontos críticos de f são as duplas (x, y) que satisfazem: 

 

x + x(2x2− 2y2− 2) = 0

(40)

Então segue da primeira expressão que devemos ter x = 0 ou 2x2 − 2y2 = 1 e da segunda

expressão devemos ter y = 0 ou 2x2− 2y2 = 3.

Daí as soluções desse sistema são os pontos (0, 0), √ 2 2 , 0 ! , − √ 2 2 , 0 !

e suas respectivas ma-trizes hessianas são:

H(0, 0) =   −2 0 0 6   e H ± √ 2 2 , 0 ! =   4 0 0 4  . Exercício 6

Dados a1, . . . , ak em Rn, determine o ponto em que a função f : Rn −→ R, definida por

f (x) =

k

X

i=1

|x − ai|2, assume o valor mínimo.

Solução. f (x) = |x − a1|2+ |x − a2|2+ · · · + |x − ak|2. Temos que ∂f ∂xi (x) = 2(hx − a1, eii + hx − a2, eii + · · · + hx − ak, eii) = 2 * kx − k X j=1 aj, ei + . Daí ∂f ∂xi (x) = 0, ∀ i = 1, . . . , n ⇔ kx − k X j=1 aj = 0 ⇔ x = k X j=1 aj k . Além disso, ∂2f ∂xj∂xi (x) = 0, se i 6= j, e ∂ 2f ∂x2 i (x) = 2k, se i = j. Desse modo Hf (x) =            2k 0 · · · 0 0 2k · · · 0 0 0 · · · 0 .. . ... . .. ... 0 0 · · · 2k            , ⇒ det Hf (x) = (2k)n > 0 ⇒ x = 1 k k X j=1 aj é ponto de mínimo de f .

Referências

Documentos relacionados